PT62.S4.Q16 - all etching tools are either pin-tipped or bladed

chinobonitochinobonito Member
edited January 2016 in Logical Reasoning 105 karma
Can y'all please help me explain this? I watched the video a few times, but the numbers method that JY explained..is just not really iron clad proof for me, or at least I'm not really understanding it.

http://7sage.com/lsat_explanations/lsat-62-section-4-question-16/

I played around with some other numbers, and that didn't really add up to the conclusion.

PT --> E
B some E
B some not E

Conclusion: #E > #not E

JY used 100 for both PT and B. That adds up and makes sense. However, I switched the numbers for PT and B to 50 and 100, respectively. So let's say some B out of 100 is 40, then 50 PT + some B (40) = 90. What's left of B thats not E are 60... 90 > 60 in conclusion, that is correct, but 50 PT and 100 B is not the same.

Thanks!

Comments

  • chinobonitochinobonito Member
    105 karma
    thinking about it, i think i was trying too hard by trying to disprove answer B. this is not a MBT question, nor is this logic game... i just need an answer that would make the premise and conclusion more valid...sufficient assumption. Anyone agree with this? Or add some more insights? Thanks!
  • nicole.hopkinsnicole.hopkins Inactive Sage Inactive ⭐
    7965 karma
    I would LOVE to help but don't have my books with me at the moment! Will circle back this evening.
  • chinobonitochinobonito Member
    105 karma
    thanks! I hope the link above works for you too if you'd like to check out the problems through there
  • nicole.hopkinsnicole.hopkins Inactive Sage Inactive ⭐
    7965 karma
    @chinobonito Ok looks like I won't get to it until the morning but will share thoughts around the time the sun rises over the North Central Texas plains ... (and hits the concrete in Dallas)
  • nicole.hopkinsnicole.hopkins Inactive Sage Inactive ⭐
    7965 karma
    I don’t have access to the Ultimate course (bills, y’all) BUT I do have this PT in front of me and have wrangled with this particular question before on several occasions.

    I’ve tried to say what I think is at issue in a number of ways, so forgive the redundancy and beating-of-a-dead-horse that goes on below. I hope at least some of it is helpful to you or at least raises further questions that can be answered more directly.

    Let’s take a look.

    All Etc are either PT or B
    Some B used for En, some are not.
    All PT used for En.
    ____________
    More Etc used for En than /En


    >>JY used 100 for both PT and B. That adds up and makes sense. However, I switched the numbers for PT and B to 50 and 100, respectively. So let's say some B out of 100 is 40, then 50 PT + some B (40) = 90. What's left of B thats not E are 60... 90 > 60 in conclusion, that is correct, but 50 PT and 100 B is not the same.

    >>However, I switched the numbers for PT and B to 50 and 100, respectively.

    What was your reasoning for doing this?

    I think there is an assumption (one that need not be made/that is not helpful or necessary) at work in the choice of these disparate quantifications:

    1) The stimulus gives no (!) indication that there are more PT or B. Given that the stimulus describes these two categories as being those to which all etching tools belong (to either one or the other), why not keeps things apples to apples (since the categories PT and B, both belong equally to the broader category of etching tools)?

    2) I believe this is the crux of your confusion: there is utterly no reason to treat PT and B as though there were more or fewer of one or the other, qua categories of objects belonging equally to the category of objects Etching tools. As they both equally belong to the sum total of all etching tools, and we are given no further information as to the distribution of Etching tools per these two categories, we ought to keep them on an even categorical playing field and quantify accordingly.

    3) It seems you may be reading the “some B [used for En, some not used for En]” as a property belonging to B at large as a subset of etching tools, which it does not—at least not in the same way. Stimulus says “All Etching tools are either Pt or B”: Does that mean that some Etching are PT and some are B? Yes—BUT. That ALL Etching are either this or that provides a conceptual framework wherein PT and B may be on entirely equal footing, numerically speaking, and we have no reason to assume they are not on equal numerical footing. I think the burden of proof falls on there being more PT or B (or rather this requires justification beyond the default). And we do not have proof that there are more PT than B, or more B than PT.

    4) And given that the stimulus is ultimately about categories of etching tool and subdivisions of said categories, it behoves us to quantify accordingly—in a conceptually useful framework, and in one that imports no extraneous assumptions not proven or needed to be proven by the stimulus. Thus, keeping numbers of PT and B themselves the same makes the most sense, and it cannot be said that 50 PT makes more sense than 100 B.

    >>So let's say some B out of 100 is 40, then 50 PT + some B (40) = 90. What's left of B thats not E are 60... 90 > 60 in conclusion, that is correct, but 50 PT and 100 B is not the same.

    What you have identified here is indeed the gap in reasoning that must be filled by an answer choice: that we are not given this numerical distribution in the stimulus. But you see how the work done above reveals that quantification as the assumption needed in order to properly draw the conclusion. Indeed we have that answer choice in B.

    That is to say: the quantification you have demonstrated cripples the argument and indeed identifies (but widens, as if this were a weaken question) that gap in reasoning. The assumption JY and I have made (and thereby provided as the gap that needs to be filled) gets us closer to the correct answer choice of B.

    Rather than quantifying in a way that demonstrates the weakness of the argument—for sufficient assumption, it’s better to quantify in a way in which the argument must be quantified in order for the conclusion to be properly drawn. IF that quantification is not given in the stimulus, then we likely must give it in an answer choice.
Sign In or Register to comment.